Đến nội dung

Hình ảnh

Topic BẤT ĐẲNG THỨC ôn thi vào lớp 10 THPT 2017 - 2018

bất đẳng thức am-gm cauchy bunyakovski minskovski schwarz holder thcs

  • Please log in to reply
Chủ đề này có 299 trả lời

#121
haccau

haccau

    Binh nhất

  • Thành viên
  • 47 Bài viết

BÀI 47:  (ĐỀ TOÁN CHUYÊN HCM-15-16)  

 Cho 2 số thực dương a,b thỏa mãn điều kiện: $a+b\leq 1$

CMR: $a^{2}-\frac{3}{4a}-\frac{a}{b}\leq -\frac{9}{4}$


Bài viết đã được chỉnh sửa nội dung bởi haccau: 05-05-2017 - 22:44

:lol:  :lol:  :lol: Don't let your dreams just be dreams!!! :lol:  :lol:  :lol: 


#122
Nguyenhuyen_AG

Nguyenhuyen_AG

    Trung úy

  • Thành viên nổi bật 2016
  • 945 Bài viết

Aảo quá .Có ai còn cách nào tự nhiên hơn không?

 

Cách dùng bất đẳng thức cổ điển thì có nhưng gõ lâu hơn phân tích bình phương.

 

Viết bất đẳng thức cần chứng minh lại như sau

\[(a^2+b^2+c^2)^3 \geqslant 9abc(a^3+b^3+c^3).\]

Giả sửa $a = \max\{a,b,c\}.$ Áp dụng bất đẳng thức AM-GM, ta có

\[9abc(a^3+b^3+c^3) \leqslant \left(ab+ca+\frac{a^3+b^3+c^3}{3a}\right)^3.\]

Ta cần chứng minh

\[a^2+b^2+c^2 \geqslant ab+ca+\frac{a^3+b^3+c^3}{3a},\]

thu gọn thành

\[\frac{(2a-b-c)(a^2+b^2+c^2-ab-bc-ca)}{3a} \geqslant 0.\]

Hiển nhiên đúng nên ta có điều phải chứng minh.

 


Nguyen Van Huyen
Ho Chi Minh City University Of Transport

#123
AnhTran2911

AnhTran2911

    Thượng sĩ

  • Thành viên
  • 230 Bài viết

BÀI 47:  (ĐỀ TOÁN CHUYÊN HCM-15-16) 

 Cho 2 số thực dương a,b thỏa mãn điều kiện: $a+b\leq 1$

CMR: $a^{2}-\frac{3}{4a}-\frac{a}{b}\leq -\frac{9}{4}$

BĐT $\Leftrightarrow\frac{a}{b}+\frac{3}{4a}\geq{a^2}+\frac{9}{4}$

Do $a+b\leq1$ nên $4ab\leq1$ và ta có đgiá $\frac{a}{b}\geq4a-4ab\geq4a-1$

Đưa về BĐT $4a+\frac{3}{4a}\geq{a^2}+\frac{13}{4}$ Đúng vì nó tương đương với $(3-a)(2a-1)^2\geq0$

Ps: Có ai làm được bài số 46 của bạn Nhoang1608 chưa ạ. Cho hỏi cái dấu bằng :v


Bài viết đã được chỉnh sửa nội dung bởi AnhTran2911: 05-05-2017 - 23:17

        AQ02

                                 


#124
Hoang Dinh Nhat

Hoang Dinh Nhat

    Sĩ quan

  • Thành viên
  • 402 Bài viết

Bài toán 46: (Albania TST 2012)

   Tìm giá trị lớn nhất của biểu thức 

                                            $\frac{1}{x^{2}-4x+9}+\frac{1}{y^{2}-4y+9}+\frac{1}{z^{2}-4z+9}$

   Trong đó $x,y,z$ là các số thực không âm thỏa mãn $x+y+z=1$.

 

Ta có bổ đề: $\frac{1}{x^2-4x+9}\leq \frac{x}{18}+\frac{1}{9}$

$\Leftrightarrow x^3-2x^2+x=x(x-1)^2\geq 0$

BĐT cuối luôn đúng nên bổ đề được chứng minh

Áp dụng, ta có: $\sum \frac{1}{x^2-4x+9}\leq \frac{x+y+z}{18}+\frac{3}{9}=\frac{7}{18}$

Đạt tại: $x=y=0;z=1$ và các hoán vị


Bài viết đã được chỉnh sửa nội dung bởi Hoang Dinh Nhat: 06-05-2017 - 09:52

Chấp nhận giới hạn của bản thân, nhưng đừng bao giờ bỏ cuộc

 

 

 

 


#125
Chu Quang Huy

Chu Quang Huy

    Binh nhì

  • Thành viên mới
  • 12 Bài viết

    Bài 48/(sưu tầm/do mình quên,đây là một bài trong đề thi chuyên của tỉnh nào đó trong mấy năm gần đây)  

          Cho các số thực a, b, c thoả : 0abc1.

Tìm giá trị lớn nhất của biểu thức: Q=a2(bc)+b2(cb)+c2(1c)

 

    Bài 49/ (đề thi vào chuyên Toán chuyên Phan Bội Châu) 

         Cho các số thực a, b, c thoả mãn a,b≥0 ; c≥1 ; a+b+c=2.

 Tìm giá trị nhỏ nhất của P = (6-a2-b2-c2) (2-abc)


Bài viết đã được chỉnh sửa nội dung bởi Chu Quang Huy: 06-05-2017 - 09:26


#126
Chu Quang Huy

Chu Quang Huy

    Binh nhì

  • Thành viên mới
  • 12 Bài viết

tích cực lên nào mấy bạn. mình thấy topic vắng vẻ quá.



#127
NHoang1608

NHoang1608

    Sĩ quan

  • Thành viên
  • 375 Bài viết

Bài toán 50 (HELLO IMO 2007).

Cho các số thực không âm $a,b,c$. Chứng minh rằng

                                  $abc+2(a^{2}+b^{2}+c^{2})+8 \geq 5(a+b+c)$.


The greatest danger for most of us is not that our aim is too high and we miss it, but that it is too low and we reach it.

----- Michelangelo----


#128
AnhTran2911

AnhTran2911

    Thượng sĩ

  • Thành viên
  • 230 Bài viết

Bài toán 50 (HELLO IMO 2007).

Cho các số thực không âm $a,b,c$. Chứng minh rằng

                                  $abc+2(a^{2}+b^{2}+c^{2})+8 \geq 5(a+b+c)$.

$LHS-RHS=\frac{1}{2} ((a^2+b^2+c^2+2abc+1-2ab-2bc-2ca)+(a+b+c-3)^2+2\sum(a-1)^2)\geq0$

PPS: Đây là hệ quả trực tiếp của BĐT tổng quát thầy Trần Nam Dũng:

Cho $a,b,c \geq0$ thì $abc+2+k[\sum(a-1)^2]\geq{a+b+c}$ Với $k$ là số thực, $k\geq\frac{1}{\sqrt2}$


Bài viết đã được chỉnh sửa nội dung bởi AnhTran2911: 06-05-2017 - 15:45

        AQ02

                                 


#129
AnhTran2911

AnhTran2911

    Thượng sĩ

  • Thành viên
  • 230 Bài viết

Bái Toán 51 ( VÕ QUỐC BÁ CẨN )

Cho $a,b,c \geq0$ thỏa mãn không có hai số nào đồng thời bằng 0 

CMR; $\sum\frac{1}{\sqrt{3a^2+bc}}\geq\frac{2+\sqrt3}{\sqrt{3(ab+bc+ca)}}$


        AQ02

                                 


#130
Mr Cooper

Mr Cooper

    Sĩ quan

  • Thành viên
  • 496 Bài viết

$\boxed{\textbf{Bài Toán 52}}$ $\text{[Vasile Cirtoaje]}$ Cho $3$ số thực dương $a,b,c$. Chứng minh rằng:

\[(a^2+b^2+c^2)^2 \ge 3(a^3b+b^3c+c^3a)\]



#131
Hoang Dinh Nhat

Hoang Dinh Nhat

    Sĩ quan

  • Thành viên
  • 402 Bài viết

$\boxed{\textbf{Bài Toán 52}}$ $\text{[Vasile Cirtoaje]}$ Cho $3$ số thực dương $a,b,c$. Chứng minh rằng:

\[(a^2+b^2+c^2)^2 \ge 3(a^3b+b^3c+c^3a)\]

 

BĐT cần chứng minh $\Leftrightarrow \frac{(3a^2-3ab-3ac-3b^2+6bc)^2+(-3ab+6ac+3b^2-3bc-3c^2)^2+(-3a^2+6ab-3ac-3bc+3c^2)^2}{18}$$\geq 0$

BĐT cuối luôn đúng nên BĐT được chứng minh


Bài viết đã được chỉnh sửa nội dung bởi Hoang Dinh Nhat: 08-05-2017 - 16:29

Chấp nhận giới hạn của bản thân, nhưng đừng bao giờ bỏ cuộc

 

 

 

 


#132
AnhTran2911

AnhTran2911

    Thượng sĩ

  • Thành viên
  • 230 Bài viết

Bài này anh huyện đã giải bằng CS. ngoài ra còn có pp phân bình phương , dùng BĐT quen thuôc hay pp tịnh tiến trên trục số và đưa về hàm bậc hai,...

File gửi kèm  Ve bai toan British MO 1986.pdf   159.45K   121 Số lần tải ( Nguyễn Văn Huyện )

P\s: có ai làm đc bài 51 chưa ạ


Bài viết đã được chỉnh sửa nội dung bởi AnhTran2911: 08-05-2017 - 16:33

        AQ02

                                 


#133
sharker

sharker

    Sĩ quan

  • Thành viên
  • 301 Bài viết

$\boxed{\textbf{Bài Toán 52}}$ $\text{[Vasile Cirtoaje]}$ Cho $3$ số thực dương $a,b,c$. Chứng minh rằng:

\[(a^2+b^2+c^2)^2 \ge 3(a^3b+b^3c+c^3a)\]

Đặt $b=a+x$, $c=a+y$ Bất đẳng thức viết laị dưới dạng :

$(x^2-xy+y^2)a^2-(x^3-5x^2y+4xy^2+y^3)a+x^4-3x^3y+2x^2y^2+y^4\geqslant 0$

TH1: $x^2-xy+y^2=0$ thì $x=y=0$ nên BDT đúng

TH2: $x^2+xy+y^2>0$

Ta có $\Delta a= (x^3-5x^2y+4xy^2+y^3)^2-4(x^2-xy+y^2)(x^4-3x^3y+2x^2y^2+4y^4)$

$=-3(x^3-x^2y-2xy^2+y^3)^2 \leqslant 0 $

Vậy Bdt được chứng minh theo định lý về dấu của tam thức bậc 2


Bài viết đã được chỉnh sửa nội dung bởi sharker: 09-05-2017 - 02:52

Anh sẽ vẫn bên em dù bất cứ nơi đâu

Anh sẽ là hạt bụi bay theo gió

Anh sẽ là ngôi sao trên bầu trời phương Bắc

Anh không bao giờ dừng lại ở một nơi nào

Anh sẽ là ngọn gió thổi qua các ngọn cây

Em sẽ mãi mãi đợi anh chứ ??

will you wait for me forever


#134
Mr Cooper

Mr Cooper

    Sĩ quan

  • Thành viên
  • 496 Bài viết

$\boxed{\textbf{Bài Toán 52}}$ $\text{[Vasile Cirtoaje]}$ Cho $3$ số thực dương $a,b,c$. Chứng minh rằng:

\[(a^2+b^2+c^2)^2 \ge 3(a^3b+b^3c+c^3a)\]

Cách khác ngắn gọn hơn

 

$(a^2+b^2+c^2)^2 \ge 3(a^3b+b^3c+c^3a)$

$\Leftrightarrow [\sum (a^2+bc-ab)]^2 \ge 3\sum(a^2+bc-ab)(b^2+ca-bc)$

Đặt $x=a^2+bc-ab;y=b^2+ca-bc;z=c^2+ab-ca$ ta quy về bài toán quen thuộc:

$(x+y+z)^2 \ge 3(xy+yz+zx) \Leftrightarrow (x-y)^2+(y-z)^2+(z-x)^2 \ge 0$



#135
Drago

Drago

    Sĩ quan

  • Thành viên
  • 462 Bài viết

Bài toán 53:[Võ Quốc Bá Cẩn]

Cho a,b,c là các số thực thoả mãn $a+b+c=6;a^2+b^2+c^2=14$.

Chứng minh rằng: $4a+b\geq 2c$


Bài viết đã được chỉnh sửa nội dung bởi Drago: 15-05-2017 - 17:42

$\mathbb{VTL}$


#136
tienduc

tienduc

    Thiếu úy

  • Điều hành viên THCS
  • 580 Bài viết

$\boxed{\textbf{Bài Toán 52}}$ $\text{[Vasile Cirtoaje]}$ Cho $3$ số thực dương $a,b,c$. Chứng minh rằng:

\[(a^2+b^2+c^2)^2 \ge 3(a^3b+b^3c+c^3a)\]

Cách khác

Xét hiệu 

$(a^2+b^2+c^2)^2-3(a^3b+b^3c+c^3a)=\frac{1}{2}(a^2-b^2+2bc-ab-ac)^2+\frac{1}{2}(b^2-c^2+2ca-bc-ab)^2+\frac{1}{2}(c^2-a^2+2ab-ca-bc)^2 \geq 0$

Từ đây ta có $đpcm$


Bài viết đã được chỉnh sửa nội dung bởi tienduc: 10-05-2017 - 21:36


#137
canletgo

canletgo

    Sĩ quan

  • Thành viên
  • 389 Bài viết

Mọi người làm hộ mk bài này với...cảm ơn nhiều  :)

Cho a, b, c và a + b + c $\leq$ 1

CMR: $\sqrt{a^{2}+\frac{1}{a^{2}}}+\sqrt{b^{2}+\frac{1}{b^{2}}}+\sqrt{c^{2}+\frac{1}{c^{2}}}\geq \sqrt{82}$


Bài viết đã được chỉnh sửa nội dung bởi canletgo: 10-05-2017 - 21:03

Alpha $\alpha$ 


#138
tienduc

tienduc

    Thiếu úy

  • Điều hành viên THCS
  • 580 Bài viết

Mọi người làm hộ mk bài này với...cảm ơn nhiều  :)

Cho a, b, c và a + b + c $\leq$ 1

CMR: $\sqrt{a^{2}+\frac{1}{a^{2}}}+\sqrt{b^{2}+\frac{1}{b^{2}}}+\sqrt{c^{2}+\frac{1}{c^{2}}}\geq \sqrt{82}$

Áp dụng BĐT $Bunyakovsky$ ta có

$\sqrt{(1^2+9^2)(a^2+\frac{1}{a^2})}\geq a+\frac{9}{a}\rightarrow a^2+\frac{1}{a}\geq \frac{1}{\sqrt{82}}(a+\frac{9}{a})$

CMTT $\rightarrow b^2+\frac{1}{b^2}\geq \frac{1}{\sqrt{82}}(b+\frac{9}{b});c^2+\frac{1}{c^2}\geq \frac{1}{\sqrt{82}}(c+\frac{9}{c})$

$\rightarrow VT\geq \frac{1}{\sqrt{82}}(a+b+c+\frac{9}{a}+\frac{9}{b}+\frac{9}{c})=\frac{1}{\sqrt{82}}[(81a+\frac{9}{a})+(81b+\frac{9}{b})+(81c+\frac{9}{c})-80(a+b+c)]$

Áp dụng BĐT $Cauchy$ ta có

$81a+\frac{9}{a}\geq 54;81b+\frac{9}{b}\geq 54;81c+\frac{9}{c}\geq 54$

$\rightarrow VT\geq \frac{1}{\sqrt{82}}(54+54+54-80)= \sqrt{82}$

Dấu $"="$ xảy ra khi $a=b=c=\frac{1}{3}$



#139
Nguyenphuctang

Nguyenphuctang

    Sĩ quan

  • Banned
  • 499 Bài viết

Hôm nay dạo trên facebook thấy bài này vui vui:
Bài 54: [Trần Hoàng Nam]

Cho a, b, c  là các số thực thỏa mãn $a+b+c=4$. Chứng minh rằng:

$$ (a^{2}+3)(b^{2}+3)(c^{2}+3) +26abc \geq 143 $$

Đẳng thức xảy ra khi nào?


Bài viết đã được chỉnh sửa nội dung bởi Nguyenphuctang: 11-05-2017 - 13:35


#140
Doflamingo

Doflamingo

    Hạ sĩ

  • Thành viên
  • 82 Bài viết

BÀI 55 (sưu tầm)

Cho x,y,z là 3 số dương thỏa mãn: 1+x+y+z=2xyz

Tìm GTNN của P= $\frac{xy}{1+x+y}+\frac{yz}{1+y+z}+\frac{zx}{1+z+x}$ 


Bài viết đã được chỉnh sửa nội dung bởi Doflamingo: 12-05-2017 - 14:05






Được gắn nhãn với một hoặc nhiều trong số những từ khóa sau: bất đẳng thức, am-gm, cauchy, bunyakovski, minskovski, schwarz, holder, thcs

1 người đang xem chủ đề

0 thành viên, 1 khách, 0 thành viên ẩn danh